Questions tagged [diophantine-approximation]

Filter by
Sorted by
Tagged with
1 vote
1 answer
230 views

Is it possible to approximate irrational by fractions with denominator and numerator odd? [closed]

Suppose $\alpha$ is a positive irrational, and $\epsilon$ is an arbitrary positive real, are there $m,n$(non-negative integers) such that $$|\alpha-(2m+1)/(2n+1)|<\epsilon/(2n+1)?$$ If they exist, ...
stephenkk's user avatar
  • 191
2 votes
0 answers
194 views

A high dimensional generation of Dirichlet approximation theorem, linear case and nonlinear case

I am working with something on Diophantine approximation, and I found a high dimensional generation of Dirichlet approximation theorem which may be true; I will be very happy if this is true. The ...
Hu xiyu's user avatar
  • 697
4 votes
0 answers
185 views

Asymptotic formula, polynomial, irrational number and uniformly distribution

Problem 1 Given a irrational number $\alpha$ and two polynomials with positive integer coefficients $P(n),Q(n)$, is it possible to get the asymptotic estimate and reasonable error term for: $$\...
Hu xiyu's user avatar
  • 697
-2 votes
1 answer
165 views

Inverse to a Diophantine number [closed]

A number $\alpha$ is said to satisfy the Diophantine condition with exponent $\beta$ iff for some constant $C>0$ the estimate $$ \left| \alpha - \frac{p}{q} \right| > \frac{C}{q^{2+\beta}} $$ ...
Anton Galaev's user avatar
7 votes
0 answers
450 views

Diophantine approximation of algebraic number

An important theorem in Diophantine approximation is the theorem of Liouville: Liouville Theorem If x is a algebraic number of degree $n$ over the rational numbers then there exists a constant c(x) ...
Hu xiyu's user avatar
  • 697
2 votes
1 answer
443 views

Counting algebraic points of bounded height

Let $K$ be a number field and $X\hookrightarrow\mathbb P^n_K$ be a projective variety of degree $\delta$ (with respect to universal bundle) and dimension $d$. We denote the set $$S(X;D,B)=\{\xi\in X(\...
var's user avatar
  • 403
114 votes
4 answers
25k views

Is the series $\sum_n|\sin n|^n/n$ convergent?

Problem. Is the series $$\sum_{n=1}^\infty\frac{|\sin(n)|^n}n$$convergent? (The problem was posed on 22.06.2017 by Ph D students of H.Steinhaus Center of Wroclaw Polytechnica. The promised prize for ...
Lviv Scottish Book's user avatar
2 votes
0 answers
108 views

Counting rare events in Diophantine approximation - distributional estimates?

Is there a sort of distributional estimate in Diophantine approximation which allows to estimate the number of solutions which provide a certain quality of approximation? For example, how large is the ...
Kurisuto Asutora's user avatar
9 votes
1 answer
260 views

Cover of the positive real numbers by intervals

For which real numbers $x$ and $y$ does the following hold?: $$ \bigcup_{\frac{a}{b} \in \mathbb{Q}^+} \left[\frac{a}{b},\frac{a}{b}+\frac{1}{a^x b^y}\right] \ = \ \mathbb{R}^+ $$
Stefan Kohl's user avatar
  • 19.5k
1 vote
0 answers
129 views

$L^\infty$ norm lower bound for Integer points in null spaces of recursively defined integer vectors?

Letting $\otimes$ be matrix kronecker/tensor product with $n\in\Bbb N$ as a parameter define non-negative integer vectors recursively $$v_1=\begin{bmatrix}a_1&b_1\end{bmatrix}\in\Bbb Z_{>0}^2$$ ...
Turbo's user avatar
  • 13.7k
2 votes
1 answer
97 views

Limits of a quasiperiodic function with two pseudoperiods

Let $\beta$ be a real number such that $\beta^2\notin\mathbb{Q}$. For any smooth function $f$ on $\mathbb{R}$ that decreases sufficiently at infinity, for example a Gaussian function, let us define $$ ...
Sylvain Ribault's user avatar
10 votes
1 answer
226 views

Distribution of good diophantine approximations

Let $\langle x \rangle: \mathbb{R} \to (-1/2,1/2]$ be the periodic function with period $1$ which is $x$ for $x \in (-1/2,1/2]$. Is there some function $D(a,b)$ of real numbers $a<b$ such that, for ...
David E Speyer's user avatar
6 votes
0 answers
198 views

How large are good approximations to irrational numbers?

It is well known that for almost every $c \in \mathbb{R} / \mathbb{Z}$ there exists $(q_n)_{n \geq 1}$ and $(a_n)_{n \geq 1}$ such that $$|c - a_n / q_n| \leq 1/ q_n^2,$$ where $q_n < q_{n+1} \leq ...
George Shakan's user avatar
11 votes
4 answers
446 views

Sequential addition of points on a circle, optimizing asymptotic packing radius

Suppose I have to put $N$ points $x_1, x_2, \ldots, x_N$ on the circle $S^1$ of length 1 so as to achieve the largest minimum separation (packing radius). The optimal solution is the equally spaced ...
Yoav Kallus's user avatar
  • 5,926
6 votes
2 answers
461 views

A naive diophantine approximation question

Let $\alpha$ be a positive real number (bigger than one, and irrational if it matters) (the one I am secretly thinking of is $\varphi,$ the golden ratio). I want to know, given an $\epsilon>0,$ ...
Igor Rivin's user avatar
  • 95.6k
8 votes
2 answers
373 views

Growth of a linear recurrent sequence

Consider the sequence defined by $a_0=a_1=1$ and $a_n=2a_{n-1}-3a_{n-2}$ for $n\geq 2$. This is the sequence https://oeis.org/A087455. I would like to prove that $|a_n|>100$ when $n>10$. How ...
Friedrich's user avatar
  • 183
2 votes
0 answers
100 views

Zero digits of a p-adic algebraic number

This question might be too simple and I just don't see something very obvious, so I apologize in advance if that is so. Let $p$ be a prime number and let $\mathbb Z_p$ denote the ring of $p$-adic ...
Anton's user avatar
  • 1,573
2 votes
0 answers
260 views

What are the best current bounds on $\times a \times b$?

Let $a,b \in \mathbb{N}_{\ge 2}$ be two integers that are multiplicatively independent (i.e., are not powers of the same integer). I have seen (Bourgain, Lindenstrauss, Michel, Venkatesh: Some ...
john mangual's user avatar
  • 22.6k
2 votes
1 answer
225 views

Bounds on near-zero integer linear combinations of numbers linearly independent over $\mathbb{Q}$

Let $\alpha_1, \alpha_2, \dots$ be an infinite sequence of real numbers such that any finite subset is linearly independent over $\mathbb{Q}$. Let $f(N)$ be the number of tuples $(m_1, \dots, m_N)$ ...
Izaak Meckler's user avatar
10 votes
1 answer
470 views

Simultaneous Diophantine approximation of $\sqrt{2}$ and $\sqrt{2\pm \sqrt{3}}$

By using the LLL algorithm, I tried to find the best simultaneous Diophantine approximation of the three numbers $\sqrt{2} $ and $ \sqrt{2 \pm \sqrt{3}} $. I was expecting that to get a precision of $\...
S. Kohn's user avatar
  • 265
0 votes
0 answers
123 views

Simultaneous Diophantine approximation in the non-generic case

Suppose we have $n$ irrational numbers $\{ x_1, x_2, \ldots, x_n \}$. For a generic set of such numbers, we have the well-known theorem that there exist infinitely many integers $q$ such that $$ \...
S. Kohn's user avatar
  • 265
3 votes
0 answers
334 views

On discrepancy of integer sequences related to Erdos-Turan-Koksma

Assume we have a sequence $a_1,\dots,a_k\in\Bbb N$ with each $a_i\approx n$ where $n$ is some integer. Suppose there exists an $\eta\in(0,1)$ such that for every $d_1,\dots,d_k\in\Bbb Z$ with $$...
user avatar
1 vote
2 answers
508 views

Beyond Dirichlet's approximation theorem

I haven't taken any number theory courses but out of curiosity I learned about Dirichlet's approximation theorem. Afterwards, it occurred to me to define the following function $f$ using 'optimal' ...
Aidan Rocke's user avatar
  • 3,827
39 votes
1 answer
2k views

Diophantine equation for 2016: triangular $|{\rm GL}_2({\bf F}_q)|$

For a prime power $q$ the group ${\rm GL}_2({\bf F}_q)$ has $(q^2-1)(q^2-q)$ elements. This happens to be a triangular number for $q=2$ (being $6 = 1+2+3$), and $-$ more notably, especially this year ...
Noam D. Elkies's user avatar
12 votes
2 answers
809 views

Are there any solutions to the diophantine equation $x^n-2y^n=1$ with $x>1$ and $n>2$?

This problem arose when considering storage of cannonballs in n-dimensional pirate ships, as explained in this MSE post. This MO question can also be reduced to the $n=3$ case. If $x,y$ is a solution ...
Sophie's user avatar
  • 223
4 votes
1 answer
199 views

Proving that $(\omega(c_1 x_1^n+\cdots+c_kx_k^n))_{n\ge 1}$ is bounded only if $|x_1|=\cdots=|x_k|$ by the Subspace Theorem

Let $c_1, \ldots, c_k \in \mathbf N^+$ and $x_1,\ldots,x_k \in \mathbf Z \setminus \{0\}$. It is possible to prove by elementary means that $(\omega(c_1 x_1^n+\cdots+c_kx_k^n))_{n\ge 1}$ is a bounded ...
Salvo Tringali's user avatar
3 votes
0 answers
154 views

Square summable sequences associated to Pisot numbers

Given a real number $x$, let $\Vert x\Vert=[x]-x$, where $[x]$ is the nearest integer to $x$. Suppose $\lambda>1$ is a Pisot number. Let $f(x)=x^k+a_{k-1}x^{k-1}+\ldots+a_0$ be the irreducible ...
Gabe Conant's user avatar
  • 3,204
2 votes
0 answers
113 views

Inhomogeneous approximation mod p

I am interested in the following discrete analog of Kronecker's inhomogeneous approximation theorem. Given $x_1,\ldots,x_n$ distinct residue classes modulo a prime $p$, and further residue classes $...
qazwsx's user avatar
  • 21
3 votes
0 answers
111 views

asymptotic behavior of $N_{\mathbb{Q(\alpha)}/\mathbb{Q}} (1-\alpha^n)$

We have an algebraic complex number $\alpha$ such that $|\alpha|=1$ and it is not a root of unity, We also know that $\alpha \in \mathcal{O}_{\mathbb{Q}(\alpha)}$ (ring of integers). I need to study ...
matimon's user avatar
  • 31
3 votes
1 answer
306 views

Vojta's conjecture on the bounded degree algebraic points over projective line?

I want to know the accurate version of the Vojta's conjecture on the bounded degree algebraic points over projective line, which is known as an extension of the Roth's theorem for bounded degree ...
Sajad Salami's user avatar
3 votes
0 answers
147 views

Metric extensions of Littlewood's conjecture

Littlewood's conjecture on simultaneous rational approximation to a pair of real numbers, $$ \liminf_{n \in \mathbb{N}} \, n \cdot \mathrm{dist}(n\alpha,\mathbb{Z}) \cdot \mathrm{dist}(n\beta, \mathbb{...
Vesselin Dimitrov's user avatar
8 votes
0 answers
250 views

Irrationality measure of the number is itself

Does there exist real number $\theta\in \mathbb{R}$\ $\mathbb{Q}$ such that Irrationality Measure of $\theta$ is itself? $$\forall \epsilon >0, \exists C>0, \forall(p,q)\in \mathbb{Z^2},\bigg|\...
vito-ვიტო's user avatar
10 votes
2 answers
925 views

Estimate number of solutions in the Roth's theorem

There is a fundamental theorem in Diophantine approximation : For all algebraic irrational $\alpha$ $$\displaystyle \left \lvert \alpha - \frac{p}{q} \right \rvert < \frac{1}{q^{2 + \...
vito-ვიტო's user avatar
2 votes
0 answers
106 views

Separation of linear forms

Let $r\ge5$ and $0<x_1<x_2<\dots<x_{r-1}<x_r<1$ be independent over $\mathbb Q$. Fix some $N\ge 5$ and consider the linear forms $$ f({\bf a})=\sum_{i=1}^r a_i x_i,\quad {\bf a}\in \{...
Nikita Sidorov's user avatar
1 vote
0 answers
166 views

Analog of Baker's theorem on linear combination of $\log a \log b$

Baker's theorem basically says that, given algebraic numbers $a_1,\ldots,a_n$ and $m_1,\ldots,m_n$, if there is no good reason for a linear combination $$\sum m_i\log a_i$$ to cancel, then it is ...
Ted Mao's user avatar
  • 443
8 votes
1 answer
689 views

Efficient Dirichlet approximation (continued fractions?) over a number field

Is there an efficient algorithm for Dirichlet approximation for a given (high-degree) number field and its ring of integers, perhaps analogous to the Euclidean/continued fractions algorithm for the ...
Chris Peikert's user avatar
1 vote
0 answers
561 views

Optimal Diophantine approximation of $\pi$

If the 'optimal' Diophantine approximation of $\pi$ is given by the maximum value of $M=-\log_q(\min_{\forall p \in \mathbb{N}} |\frac{p}{q}-\pi|)$ for $q \geq 2$, what is this value?
Aidan Rocke's user avatar
  • 3,827
0 votes
0 answers
95 views

Criterion for irrational numbers of constant type 2

From Kuiper's and Niederreiter's book Uniform distribution of sequences, Ch.2, § 3, I learn that an irrational number $\alpha\in \mathbf{R}\smallsetminus \mathbf{Q}$ is of constant type $\eta$ if ...
Daniel Miller's user avatar
6 votes
0 answers
210 views

Solving polynomial equations modulo $1$

Let $P\in \mathbb{R}\lbrack x\rbrack$ be given. (In practice, the coefficients could be given as, say, decimals to sufficient precision.) Let $M\geq 1$, and let $I$ be an interval in $\mathbb{R}/\...
H A Helfgott's user avatar
  • 19.4k
6 votes
1 answer
461 views

On property of monic polynomial with integer coefficients

For a monic polynomial with integer coefficients $f$ where $\partial f = 2$, we have $$ \textrm{inf}(f(x)) > 0 \implies \textrm{inf}(f(x)) \geq \frac{3}{4} . $$ Could we generalize this (for ...
Lwins's user avatar
  • 1,531
2 votes
1 answer
152 views

On the construction of a certain sequence of integers

Let $\alpha \in \mathbb{R}$ be a fixed (positive) number. For each $k \in \mathbb{N}$ we choose $\varepsilon_k >0$ with the property that $\lim_k \varepsilon_k =0$. If $\alpha \in \mathbb{R} \...
Siminore's user avatar
  • 459
2 votes
0 answers
69 views

On a sequence of integers

I recall a well-known theorem due to Minkowski. Theorem. If $\theta$ is irrational and $\alpha$ is not of the form $\alpha = m\theta+n$ for some integers $m$ and $n$, then there are infinitely many ...
Siminore's user avatar
  • 459
2 votes
1 answer
116 views

On cluster points of a particular sequence

This is the sequel of a previous question. Let us consider the sequence $$ \xi_n = 2n \{n\xi\}-n, $$ where $\xi>0$ is a given real irrational number and $\{\cdot\}$ is the fractional part. Do ...
Siminore's user avatar
  • 459
-1 votes
2 answers
223 views

Precise asymptotic of diophantine approximation

I know that every irrational number $\xi$ can be approximated by rational numbers in such a way that $$ \left| \xi - \frac{p}{q} \right| \leq \frac{c}{q^2} $$ for infinitely many choices of $p$ and $q$...
Siminore's user avatar
  • 459
7 votes
2 answers
340 views

On the density of the sequence $\{n \{n \xi \} \}_n$

I have a question that I can't manage to answer myself. It comes from some work in PDE theory, but it is related to analytic number theory. Let us say that we have an irrational number $\xi$. The ...
Siminore's user avatar
  • 459
11 votes
2 answers
469 views

Can two rational rotations $F_2 = \langle A, B \rangle \to SO(3)$ efficiently approximate the $3 \times 3$ identity matrix?

Let $A,B$ be two rational rotations: $$ A = \left[\begin{array}{rcc} \frac{3}{5} & \frac{4}{5} & 0 \\ -\frac{4}{5} & \frac{3}{5} & 0 \\ 0 & 0 & 1 \end{array}\right] \quad\...
john mangual's user avatar
  • 22.6k
4 votes
2 answers
399 views

A particular Diophantine approximation of $\pi/2$

I have asked this question in math.stackexchange without any answer, so I have decided to post it here too. Recently I was playing around with the sequence $$\frac{1}{n\sin(n)},\ n\in\mathbb{N}.$$ ...
Tomás's user avatar
  • 409
8 votes
2 answers
1k views

Approximating any integer by multiples of 2 and 3

Given any integer $n$ sufficiently large, I want to prove (or disprove) that there exists another integer $m\ge n$ with the form $m=2^a3^b$ ($a,b$ are no negative integers) such that $m-n=o(n)$, i.e., ...
lchen's user avatar
  • 436
13 votes
0 answers
305 views

Diophantine approximation in the Julia set

Let $f : \mathbb{CP}^1 \to \mathbb{CP}^1$ be a rational map of degree $q > 1$; or just a quadratic binomial $z^2 + c$, if one prefers. The Julia set $J_f$ is the closure of the repelling periodic ...
Vesselin Dimitrov's user avatar
28 votes
3 answers
2k views

Does $a_n=\prod^n_{k=1}(1-e^{k\alpha \pi i})$ converge to zero when $\alpha$ is irrational?

I came across a problem concerning about the convergence of products. I wonder if the complex series $a_n=\prod^n_{k=1}(1-e^{k\alpha \pi i})$ converges to zero when $\alpha$ is irrational. Of course, ...
No One's user avatar
  • 1,553

1 2 3
4
5
7